OG12-Qn no 112 Is there any better approach to solve

This topic has expert replies
Senior | Next Rank: 100 Posts
Posts: 62
Joined: Wed May 04, 2011 9:50 pm
Thanked: 2 times
Followed by:2 members
While approching the following question,

The average hourly wage of television assemblers in Vernland has long been significantly lower than that in neighboring Borodia. Since Borodia dropped all tariffs on Vernlandian televisions three years ago, the number of televisions sold annually in Borodia has not changed. However, recent statistics show a drop in the number of television assemblers in Borodia. Therefore, updated trade statistics will probably indicate that the number of televisions Borodia imports annually from Vernland has increased.Which of the following is an assumption on which the argument depends?

(A) The number of television assemblers in Vernland has increased by at least as much as the number of television assemblers in Borodia has decreased.

(B) Televisions assembled in Vernland have features that televisions assembled in Borodia does not have.

(C) The average number of hours it takes a Borodian television assembler to assemble a television has not decreased significantly during the past three years.

(D) The number of televisions assembled annually in Vernland has increased significantly during the past three years.

(E) The difference between the hourly wage of television assemblers in Vernland and the hourly wage of television assemblers in Borodia is likely to decrease in the next few years.

My queries are as follows,

a) I have set a time limit for this qn and i have taken this Qn, first i have read the argument and i came to a point that this qn will take lot of time. Hence, I have gone through the arguement and went to the answer choices, then i started to do Process of elimination.

First, I eliminated answer choice B,D & E. Then I kept answer choice A & C as contender.

Coming to A & C, when i applied negation test

(A) The number of television assemblers in Vernland has increased by not at least as much as the number of television assemblers in Borodia has decreased.

(C) The average number of hours it takes a Borodian television assembler to assemble a television has decreased significantly during the past three years.

In both the answer choice after applying negation test both the choice does not affect the conclusion directly. While evaluating these two choice, I found very difficult to find the correct answer choice, hence I choosed wrong answer choice A after some guessing.

When doing this qn without time limit, this Qn is a difficult one and only for splitting & analysing the arguement it is taking more than 3-4 mins.

But even with this approach it took me a long time to come to the correct answer choice C.

1)So finally, this kind of difficult Qn with lot of time consuming how we can approach the qn in a better way?

Anyone pls explain me some good strategy for this kind of time consuming difficult qn.

Master | Next Rank: 500 Posts
Posts: 138
Joined: Thu Nov 12, 2009 10:07 am
Thanked: 19 times
Followed by:3 members

by GmatVerbal » Tue Dec 13, 2011 6:03 pm
I would eliminate A- simply for the fact that is too big as assumption too make. As there is nothing we know about process/technology used in Vernland. So we don't know productivity levels of the employees there.

I am not a big fan of assumption negation technique. I would spend that time on analyzing/understanding the question stem. Normally in assumption questions some kind of scope change (jump/gap) in the logic of conclusion. So simple rephrasing and diagramming may help.

Also, in causal statements You always see one side of the picture(argument). The other side is used for strengthening / weakening /assumptions;

X and Y occurring together and so X is the reason for Y. What is missing is here Is Y causing X or Some other Z causing X; The question stem doesn't talk about Z at all.


Also one of the concepts I have seen GMAT testing in Verbal is X+Y =K; The question stem contains X & K. That is the relation ship is established between X & K and X changes K should change. But K is constant. Solve the paradox. Look for "Y"; i.e. K is also dependent on K X & Y;

Coming back to question:
The argument is;

Number of assemblers in Borodia has dropped ===> So probably updated trade statistics indicate an
increase of imports from Vernland;

There is a jump in the conclusion; Fill up the jump;

number of assemblers dropped ==> so number of TV set made in Borodia dropped ===> so its importing from Vernland;

The clear assumption is number of TV sets made in Borodia dropped; The answer should be some what close to this.

You can arrive at by asking a chain of questions?

The first question is why is that conclusion can be made? answer: Borodia must be importing more TV sets? why is that? because number of assemblers reduced. so what? This answers the puzzle.

This fits to what K = f(X,Y); K= TV sets; X = assemblers; Y = some other factors;

Just by changing X , K need not be changed as some other factors Y may be changing;

K may be different in each question: K may be profits

Profits may depend on different factors; Question stem gives only one factor;

----- Take a look at the following GMAT prep questions -----------
Q1)

Tiger sharks are common in the water surrounding Tenare island. Usually tiger sharks feed on smller sharks, but sometimes they have attacked tourists swimming and surfing at beaches. This has hurt tourism industry, which is second only to its fishing industry in annual revenues. In order to help the economy, therefore, the mayor of the island has proposed and ongoing program to kill any tiger sharks within a mile of the beach.

Which of the following, if true, most strongly calls into question the likelihood that implementation of the mayor's proposal will have the desire consequence?

A. Even if not all the tiger sharks that come to the beach are killed, the existence of the program would reassure tourists.

B. Business owners who depend on tourism are willing to pay most of the cost of implementing the program

C. Tourists come to Tenare Island especially for its beaches.

D. The small sharks on which tiger sharks prey feed on fish that are commercially important to the island's fisheries.

E. Not all tourists who come to Tenare Island enjoy swimming or surfing.

----
Q2)

Jennifer: Video rental outlets in Centerville together handled 10,000 fewer video rentals in 1994 than in 1993. The decline in rentals was probably due almost entirely to the February 1994 opening of Videorama, the first and only video rental outlet in the area that, in addition to renting videos, also sold them cheaply.

Brad: There must be another explanation: as you yourself said, the decline was on the order of 10,000 rentals. Yet Videorama sold only 4,000 videos in 1994.

Which of the following, if true, would most seriously weaken the force of the objection that Brad presents to Jennifer's explanation?

(A) In 1994 Videorama rented out more videos than it sold.
(B) In 1994 two new outlets that rent but that do not sell videos opened in Centerville.
(C) Most of the video rental outlets in Centerville rent videos at a discount on certain nights of the week.
(D) People often buy videos of movies that they have previously seen in a theater.
(E) People who own videos frequently loan them to their friends.

Senior | Next Rank: 100 Posts
Posts: 62
Joined: Wed May 04, 2011 9:50 pm
Thanked: 2 times
Followed by:2 members

by kishokbabu » Tue Dec 13, 2011 10:11 pm
GmatVerbal wrote:I would eliminate A- simply for the fact that is too big as assumption too make. As there is nothing we know about process/technology used in Vernland. So we don't know productivity levels of the employees there.

I am not a big fan of assumption negation technique. I would spend that time on analyzing/understanding the question stem. Normally in assumption questions some kind of scope change (jump/gap) in the logic of conclusion. So simple rephrasing and diagramming may help.

Also, in causal statements You always see one side of the picture(argument). The other side is used for strengthening / weakening /assumptions;

X and Y occurring together and so X is the reason for Y. What is missing is here Is Y causing X or Some other Z causing X; The question stem doesn't talk about Z at all.


Also one of the concepts I have seen GMAT testing in Verbal is X+Y =K; The question stem contains X & K. That is the relation ship is established between X & K and X changes K should change. But K is constant. Solve the paradox. Look for "Y"; i.e. K is also dependent on K X & Y;

Coming back to question:
The argument is;

Number of assemblers in Borodia has dropped ===> So probably updated trade statistics indicate an
increase of imports from Vernland;

There is a jump in the conclusion; Fill up the jump;

number of assemblers dropped ==> so number of TV set made in Borodia dropped ===> so its importing from Vernland;

The clear assumption is number of TV sets made in Borodia dropped; The answer should be some what close to this.

You can arrive at by asking a chain of questions?

The first question is why is that conclusion can be made? answer: Borodia must be importing more TV sets? why is that? because number of assemblers reduced. so what? This answers the puzzle.

This fits to what K = f(X,Y); K= TV sets; X = assemblers; Y = some other factors;

Just by changing X , K need not be changed as some other factors Y may be changing;

K may be different in each question: K may be profits

Profits may depend on different factors; Question stem gives only one factor;

----- Take a look at the following GMAT prep questions -----------
Q1)

Tiger sharks are common in the water surrounding Tenare island. Usually tiger sharks feed on smller sharks, but sometimes they have attacked tourists swimming and surfing at beaches. This has hurt tourism industry, which is second only to its fishing industry in annual revenues. In order to help the economy, therefore, the mayor of the island has proposed and ongoing program to kill any tiger sharks within a mile of the beach.

Which of the following, if true, most strongly calls into question the likelihood that implementation of the mayor's proposal will have the desire consequence?

A. Even if not all the tiger sharks that come to the beach are killed, the existence of the program would reassure tourists.

B. Business owners who depend on tourism are willing to pay most of the cost of implementing the program

C. Tourists come to Tenare Island especially for its beaches.

D. The small sharks on which tiger sharks prey feed on fish that are commercially important to the island's fisheries.

E. Not all tourists who come to Tenare Island enjoy swimming or surfing.

----
Q2)

Jennifer: Video rental outlets in Centerville together handled 10,000 fewer video rentals in 1994 than in 1993. The decline in rentals was probably due almost entirely to the February 1994 opening of Videorama, the first and only video rental outlet in the area that, in addition to renting videos, also sold them cheaply.

Brad: There must be another explanation: as you yourself said, the decline was on the order of 10,000 rentals. Yet Videorama sold only 4,000 videos in 1994.

Which of the following, if true, would most seriously weaken the force of the objection that Brad presents to Jennifer's explanation?

(A) In 1994 Videorama rented out more videos than it sold.
(B) In 1994 two new outlets that rent but that do not sell videos opened in Centerville.
(C) Most of the video rental outlets in Centerville rent videos at a discount on certain nights of the week.
(D) People often buy videos of movies that they have previously seen in a theater.
(E) People who own videos frequently loan them to their friends.
Hi

Thanks Gmatverbal,

For Q1- the answer is E

In this Qn, I am getting a dilema between A & E choices.

But Answer choice A states that mayor's proposal will have desire consequence, answer choice E question the mayor's proposal.

Hence, E is the correct answer.

For Q2- the answer is E

Jennifer's explanation.:
X- opening of videorama in 1994 which rent and sell the videos in cheap price.
Y- Video rental outlets in Centerville together handled 10,000 fewer video rentals in 1994 than in 1993

X ==> Y

Brad's point.: X causes Y but due to some K reasons other than X, because Videorama sold only 4,000 videos in 1994.

The Qn stem ask us to find that there is no K but only X and teh answer choice should support X.

Hence answer choice E which states that eventhough 4000 videos sold the purchaser loan them to their friends which provides r weakens the force of objection of Brad's statement.

@GMATVERBAL .:Kindly give your opinion for this type of analysis

Master | Next Rank: 500 Posts
Posts: 138
Joined: Thu Nov 12, 2009 10:07 am
Thanked: 19 times
Followed by:3 members

by GmatVerbal » Mon Dec 19, 2011 1:47 pm
I have just seen your post, so late reply.
Q2) is correct in your analysis.

Q1) In evaluate plan questions, pay close attention to plan goal;
Goal is to "help economy"; From the question stem economy is dependent on two factors
tourism revenues and fishing industry revenues;
Economy = tourism revenues + fishing industry revenues;

When Mayor did the plan to kill sharks he should make sure he not hurting fishing industry in one way or other;

Now you should be able to answer the question.

Senior | Next Rank: 100 Posts
Posts: 62
Joined: Fri Aug 12, 2011 12:37 am
Thanked: 3 times
Followed by:1 members

by sk8legend408 » Tue Dec 20, 2011 7:18 am
In GmatVerbal's question 1 regarding the mayor's plan the correct answer is D. The tiger sharks eat the smaller sharks, which eat the fish that are commercially important to the fisheries.

If the tiger sharks are killed, the smaller sharks will eat the fish that are commercially important to the island's fisheries. And fishing is the island's number one source of revenue. Therefore the answer is D.

User avatar
GMAT Instructor
Posts: 1035
Joined: Fri Dec 17, 2010 11:13 am
Location: Los Angeles, CA
Thanked: 474 times
Followed by:365 members

by VivianKerr » Tue Dec 20, 2011 11:26 am
Sometimes negation test is useful, but I would always extract the conclusion, evidence, and come up with your own assumption first BEFORE reading the answer choices. It will help you eliminate more quickly if you have a very strong grasp of the argument.

Conclusion: # of TVs B imports from V has increased

Evidence: Fewer assemblers of TVs in B, # of TV's sold in B has not changed

Assumption: B isn't getting TVs from elsewhere, the decline in # of B assemblers means less TV's made in B

"depends" in the question stem means the correct answer will most clearly strengthen the conclusion that the # of TVs imported from V has increased.

Our assumptions most closely matches C, since it shows that the decline in # of assemblers means less TV's made in B. What would WEAKEN would be if the B assemblers just became more efficient.
Vivian Kerr
GMAT Rockstar, Tutor
https://www.GMATrockstar.com
https://www.yelp.com/biz/gmat-rockstar-los-angeles

Former Kaplan and Grockit instructor, freelance GMAT content creator, now offering affordable, effective, Skype-tutoring for the GMAT at $150/hr. Contact: [email protected]

Thank you for all the "thanks" and "follows"! :-)